0 Daumen
1,2k Aufrufe

Brauche Hilfe bei der folgenden Aufgabe:

Ist an ≥ 0 und konvergiert die Folge (an) gegen a, so konvergiert die Folge √(an) gegen \( \sqrt{a} \)

Dies ist zu zeigen.

Avatar von

Vom Duplikat:

Titel: Zeigen Sie: Ist an ≥ 0 und konvergiert die Folge (an) gegen a, so konvergiert die Folge Wurzel a_{n} gegen Wurzel a.

Stichworte: folge

Aufgabe:

Zeigen Sie: Ist an ≥ 0 und konvergiert die Folge (an) gegen a, so konvergiert die Folge Wurzel an gegen Wurzel a.

Für a>0 hilft der bekannte "Trick":

$$\sqrt{a_n}-\sqrt{a}=(\sqrt{a_n}-\sqrt{a})\frac{\sqrt{a_n}+\sqrt{a}}{\sqrt{a_n}+\sqrt{a}}$$

Ist ja schon beantwortet:

https://www.mathelounge.de/847848/ist-konvergiert-die-folge-gegen-konvergiert-die-folge-gegen

Gruß Mathhilf

danke dir das ist wirklich die beste Seite..............

2 Antworten

0 Daumen
 
Beste Antwort

Aloha :)

Wir wissen, dass \((a_n)\to a\) konvergiert.

1. Fall \(a>0\)

Wählen wir ein \(\varepsilon>0\) beliebig, aber fest, so gilt ab einem gewissen \(n_0\in\mathbb N\):$$\left|a_n-a\right|<\varepsilon\implies\left|(\sqrt{a_n}-\sqrt a)(\sqrt{a_n}+\sqrt a)\right|<\varepsilon\implies$$$$\left|\sqrt{a_n}-\sqrt a\right|<\frac{\varepsilon}{\sqrt{a_n}+\sqrt a}\le\frac{\varepsilon}{\sqrt a}$$Daher konvergiert \((\sqrt{a_n})\to \sqrt{a}\).

2. Fall \(a=0\)

Wählen wir ein \(\varepsilon>0\) beliebig, aber fest, so gilt ab einem gewissen \(n_0\in\mathbb N\):$$\left|a_n\right|<\varepsilon\implies\left|\sqrt{a_n}\right|<\sqrt{\varepsilon}$$Daher konvergiert \((\sqrt{a_n})\to0\).

Avatar von 148 k 🚀
0 Daumen

Hallo

du musst aus |an-a|<ε1 für n>N0 ein N1 finden, so dass  |√an-√a|<ε für n>N1

Gruß lul

Avatar von 106 k 🚀

Ein anderes Problem?

Stell deine Frage

Willkommen bei der Mathelounge! Stell deine Frage einfach und kostenlos

x
Made by a lovely community